A farm tractor tows a 3300 kg trailer up a 14 degree incline with a steady speed of 2.8 m/s. What force does the tractor exert on the trailer?

Answers

Answer 1

We are asked to determine the force that the tractors is exerting on a trailer up an incline. A free-body diagram of the situation is the following:

Where:

[tex]\begin{gathered} F=\text{ force of the tractor} \\ m=\text{ mass of the trailer} \\ g=\text{ acceleratio of gravity} \end{gathered}[/tex]

Now, we add the forces in the direction of the incline:

[tex]\Sigma F_x=F-mg_x[/tex]

To determine the x-component of "mg" we use the following right triangle:

Now, we use the function sine to determine the value of "mgx":

[tex]\sin14=\frac{mg_x}{mg}[/tex]

Now, we multiply both sides by "mg":

[tex]mg\sin14=mg_x[/tex]

Now, we substitute the values of "m" and "g":

[tex](3300kg)(9.8\frac{m}{s^2})\sin14=mg_x[/tex]

Solving the operations:

[tex]7823.75N=mg_x[/tex]

Now, we substitute the value in the sum of forces:

[tex]\Sigma F_x=F-7823.75N[/tex]

Since the object is moving at a steady speed this means that the sum of forces is zero:

[tex]F-7823.75N=0[/tex]

Now, we add 7823.75N to both sides:

[tex]F=7823.75N[/tex]

Therefore, the tractor exerts a force of 7823.75N

A Farm Tractor Tows A 3300 Kg Trailer Up A 14 Degree Incline With A Steady Speed Of 2.8 M/s. What Force
A Farm Tractor Tows A 3300 Kg Trailer Up A 14 Degree Incline With A Steady Speed Of 2.8 M/s. What Force

Related Questions

A 6 kg sign is suspended by two strings making angles with the ceiling as shown in the diagram. Determine the magnitudes of the tensions in each string. Remember that 1kg exerts a force of 9.8 N. please use sine/cosine law for this question

Answers

The triangle representing the given scenario is shown below

T1 and dT2 represents the tension in each spring

We would apply the sine rule which is expressed as

a/SinA = b/SinB = c/SinC

where

A, B and C are the angles of the triangle

a, b and c are the sides opposite the respective angles

Thus,

A = 42

a = T1

B = 23

b = T2

C = 115

c = 58.8

T1/sin42 = 58.8/sin115

By cross multiplying, it becomes

T1sin115 = 58.8sin42

T1 = 58.8sin42/sin115

T1 = 43.41

T2/sin23 = 58.8/sin115

By cross multiplying, it becomes

T2sin115 = 58.8sin23

T2 = 58.8sin23/sin 115

T2 = 25.35

The magnitude of the tension in each string are 43.41 N and 25.35 N

A girl holds an arrow in her hands, ready to shoot the arrow at a dragon who is just outside her arrow’s range of 45 meters. The girl then slides down a cliffside, giving her a velocity of 8.5 m/s at an angle of 45 below the horizontal. She points her bow 30 degrees above the horizontal and shoots, the arrow leaves the bow with a velocity of 45 m/s. The girl is 55 meters away from the dragon horizontally. How far above the girl is the dragon? (Assuming the arrow hits.)I already figured out how long it takes for the arrow to reach the dragon (4.72 seconds) but I don’t know how to calculate the distance between the girl and the dragon. Please help!

Answers

First, assume that the arrow leaves the bow with a velocity of 45 m/s above the horizontal with respect to the bow.

Since the bow is moving at 8.5 m/s 45º below the horizontal, find the initial velocity of the arrow with respect to the ground:

[tex]\vec{v}_{AG}=\vec{v}_{AB}+\vec{v}_{BG}[/tex]

This equation reads:

The velocity of the arrow with respect to the ground is equal to the velocity of the arrow with respect to the bow plus the velocity of the bow with respect to the gound.

Notice that this is a vector equation. Then, the vertical and horizontal components of the velocities must be added separately:

[tex]\begin{gathered} v_{AG-x}=v_{AB-x}+v_{BG-x} \\ v_{AG-y}=v_{AB-y}+v_{BG-y} \end{gathered}[/tex]

Find the vertical and horizontal components of the velocity of the arrow with respect to the bow and the velocity of the bow with respect to the ground:

[tex]\begin{gathered} v_{AB-x}=v_{AB}\cos (\theta) \\ =45\frac{m}{s}\cdot\cos (30º) \\ =38.97\frac{m}{s} \end{gathered}[/tex]

[tex]\begin{gathered} v_{AB-y}=v_{AB}\sin (\theta) \\ =45\frac{m}{s}\sin (30º) \\ =22.5\frac{m}{s} \end{gathered}[/tex]

Similarly, for the velocity of the bow with respect to the ground:

[tex]\begin{gathered} v_{BG-x}=6.01\frac{m}{s} \\ v_{BG-y}=-6.01\frac{m}{s} \end{gathered}[/tex]

Then, the vertical and horizontal components of the initial velocity of the arrow with respect to the ground, are:

[tex]\begin{gathered} v_{AG-x}=38.97\frac{m}{s}+6.01\frac{m}{s}=44.98\frac{m}{s} \\ \\ v_{AG-y}=22.5\frac{m}{s}-6.01\frac{m}{s}=16.49\frac{m}{s} \end{gathered}[/tex]

Use the horizontal component of the velocity to find how long it takes for the arrow to travel a horizontal distance x of 55 meters. Then, use that time to find the vertical position of the arrow.

Since the horizontal movement of the arrow is uniform, then:

[tex]v_{AG-x}=\frac{x}{t}_{}[/tex]

Isolate t and substitute x=55m, v_{AG-x}=44.98 m/s:

[tex]\begin{gathered} t=\frac{x}{v_{AG-x}} \\ =\frac{55m}{44.98\frac{m}{s}} \\ =1.2227s \end{gathered}[/tex]

The vertical motion of the arrow is a uniformly accelerated motion. Then, the vertical position is given by:

[tex]y=v_{AG-y}t-\frac{1}{2}gt^2[/tex]

Replace v_{AG-y}=16.49 m/s, t=1.2227s and g=9.81 m/s^2 to find the vertical position of the arrow when the horizontal position is 55 meters. This matches the elevation of the dragon with respect to the girl when the girl shoots:

[tex]\begin{gathered} y=(16.49\frac{m}{s})(1.2227s)-\frac{1}{2}(9.81\frac{m}{s^2})(1.2227s)^2 \\ =12.829\ldots m \\ \approx12.8m \end{gathered}[/tex]

Therefore, the dragon is 12.8 meters above the girl when the arrow is shoot.

what is this math problem Solve for x.

0.5x = 4.5

Answers

x = 9

if you format it as a fraction, it’s easier to solve.

1/2 * x = 9/2
divide both sides by 1/2

x = 9/2 * 2/1
x = 9
Answer: x = 9

Explanation: 0.5x = 4.5

Divide both sides: 4.5/0.5

Multiply numerator and denominator: x = 45/5

Cross out common factor: x = 9

which Vector goes from 5, 5 to 4, 0

Answers

The required vector is  î + 5ĵ

Those physical quantities that have magnitude, as well as direction, are represented by a Vector.  Some examples of Vector quantities are Force, Acceleration, Velocity, and Torque.

According to the given information, we represent the points in vector form,

The 1st vector is [tex]R_{1}[/tex] = 5î + 5ĵ

And the Second Vector is [tex]R_{2}[/tex] = 4î

Now, it is given that the required vector goes from 5,5 to 4,0.

So, the required vector is R,

Thus, R = (5-4)î + (5-0)ĵ

R = î + 5ĵ

Hence, the required vector is  î + 5ĵ

To read more about vectors, visit https://brainly.com/question/13322477

#SPJ1

Answer:

Vector B

Explanation:

An object is placed 3.00 cm in front of a concave lens with a focal length of 5.00 cm. Find the image distance.

Answers

ANSWER:

-1.88 cm

STEP-BY-STEP EXPLANATION:

The situation would be the following:

Given:

Object distance (u): -3 cm

Focal distance (f): -5 cm

Using the lens formula, we calculate the distance of the image:

[tex]\begin{gathered} \frac{1}{v}-\frac{1}{u}=\frac{1}{f} \\ \\ \text{We replacing} \\ \\ \frac{1}{v}-\frac{1}{-3}=\frac{1}{-5} \\ \\ \frac{1}{v}\cdot \:15v-\frac{1}{-3}\cdot \:15v=\frac{1}{-5}\cdot \:15v \\ \\ 15+5v=-3v \\ \\ 5v=-3v-15 \\ \\ 8v=-15 \\ \\ v=-\frac{15}{8} \\ \\ v=-1.88\text{ cm} \end{gathered}[/tex]

The image distance is -1.88 cm

Answer:

Explanation:

yfdyrdrd

Which statement describes the sign of gravitational force?(1 point)

A Gravitational force is negative because it is attractive.
B Gravitational force is negative because it is repulsive.
C Gravitational force is positive because it is attractive.
D Gravitational force is positive because it is repulsive

Answers

The statement that describes the sign of gravitational force is as follows: Gravitational force is negative because it is attractive (option A).

What is gravitational force?

Gravitational force is a very long-range, but relatively weak fundamental force of attraction that acts between all particles that have mass. It is believed to be mediated by gravitons.

According to the Newton’s law of gravitation, which explains the gravitational force, every particle in the universe attracts every other particle with a force that is directly proportional to the product of their masses and inversely proportional to the square of the distance between them.

The negative sign in the gravitational force equation indicates that the gravitational force interaction is always attractive.

Learn more about gravitational force at: https://brainly.com/question/12528243

#SPJ1

A ball is moving at 4 m/s and has momentum 48 kg.m/s. What is the ball's mass (in kilograms)?

Answers

Given:

The speed of the ball is v = 4 m/s

The momentum of the ball is p = 48 kg m/s

Required: The mass of the ball

Explanation:

The mass of the ball can be calculated as

[tex]\begin{gathered} p=mv \\ m=\frac{p}{v} \\ m=\frac{48\text{ kg m/s}}{4\text{ m/s}} \\ =12\text{ kg} \end{gathered}[/tex]

Final Answer: The mass of the ball is 12 kg.

Yea I think and this the other day and

Answers

Given:

The radius of the tank is: R = 7.5 feet.

Volume rate is: dV/dt = 60 cubic feet per hour.

h = 2.8 feet.

To find:

V, R, h, dV/dt, dR/dt, dh/dt

Explanation:

The given expression for volume is:

[tex]V=\pi h^2(R-\frac{1}{3}h)[/tex]

Substituting the values in the above equation, we get:

[tex]V=3.14\times2.8^2\times(7.5-\frac{1}{3}\times2.8)=161.74\text{ cubic feet}[/tex]

The volume is 161.74 cubic feet.

The radius of the tank is given as: R = 7.5 feet

The height of the water in the tank is given as: h = 2.8 feet

The volume rate of water is given as: dV/dt = 60 cubic feet per hour

As the radius of the tank is constant, its derivative will be zero. Thus, dR/dt = 0.

dh/dt can be calculated by differentiating the given volume equation with respect to time as:

[tex]\begin{gathered} \frac{dV}{dt}=\pi^2\times\frac{dh^2}{dt}\times(R-\frac{1}{3}\frac{dh}{dt}) \\ \\ \frac{dV}{dt}=\pi^2\times\frac{2dh}{dt}\times(R-\frac{1}{3}\frac{dh}{dt}) \\ \\ \frac{dV}{dt}=2\pi^2R\times\frac{dh}{dt}-\frac{2\pi^2}{3}\times(\frac{dh}{dt})^2 \end{gathered}[/tex]

Substituting the values in the above equation, we get:

[tex]\begin{gathered} 60=2\pi^2\times7.5\times\frac{dh}{dt}-\frac{2\pi^2}{3}\times(\frac{dh}{dt})^2 \\ \\ 60=148.044\frac{dh}{dt}-6.580(\frac{dh}{dt})^2 \\ \\ 6.580(\frac{dh}{dt})^2-148.044\frac{dh}{dt}+60=0 \end{gathered}[/tex]

Solving the above quadratic equation, we get:

dh/dt = 0.412 ft/hr or dh/dt = 22.086 ft/hr.

Final answer:

V = 161.74 cubic feet

R = 7.5 feet

h = 2.5 feet

dV/dt = 60 cubic feet per hour

dR/dt = 0

dh/dt = 22.086 ft/hr or 0.412 ft/hr

In a tug-of-war two teams are pulling in opposite directions, but neither team ismoving. What do the net forces equal in this example?

Answers

so, the net forces equal zero

Explanation

Step 1

Draw the situation

Newton's first law says that if the net force on an object is zero, then that object will have zero acceleration

in this case due to neither team is moving, we can say the center point is at rest,so its acceleration is zero

According to Newton's law, as the object is at rest, the sum of the forces acting on it equals zero

so

sum of forces = 0

the forces are due to the teams pulling so,

[tex]\begin{gathered} \text{Force team 1 -force team 2= 0} \\ \text{if we move the second term to the rigth} \\ \text{Force team 1=Force team }2 \end{gathered}[/tex]

so, the net forces equal zero

You have a light spring which obeys Hooke's law. This spring stretches 3.02 cm vertically when a 2.50 kg object is suspended from it. Determine the following.(a) the force constant of the spring (in N/m)N/m(b) the distance (in cm) the spring stretches if you replace the 2.50 kg object with a 1.25 kg objectcm(c) the amount of work (in J) an external agent must do to stretch the spring 8.90 cm from its from unstretched positionJ

Answers

We are given that a spring stretches 3.02 cm vertically when a 2.5 kg object is suspended.

Part (a) To determine the constant of the spring we need first to determine the weight of the object, to do that we will use the following formula:

[tex]W=mg[/tex]

Where:

[tex]\begin{gathered} m=\text{ mass} \\ g=\text{ acceleration of gravity} \end{gathered}[/tex]

Now we plug in the values:

[tex]W=(2.5\operatorname{kg})(9.8\frac{m}{s^2})[/tex]

Solving the operations we get:

[tex]W=24.5N[/tex]

Now we use Hooke's law:

[tex]F=kx[/tex]

Where:

[tex]\begin{gathered} F=\text{ force} \\ k=\text{ spring contant} \\ x=\text{ distance stretched} \end{gathered}[/tex]

Now we solve for "k" by dividing both sides by "x":

[tex]\frac{F}{x}=k[/tex]

Now, since the object is placed vertically this means that the only force acting on the spring in the weight of the object, therefore:

[tex]F=W[/tex]

Now we plug in the known values:

[tex]\frac{24.5N}{3.02\operatorname{cm}}=k[/tex]

Solving the operations we get:

[tex]8.11\frac{N}{\operatorname{cm}}=k[/tex]

Since we are required to express the constant in N/m, we need to convert the centimeters into meters. To do that we will use the following conversion factor:

[tex]100\operatorname{cm}=1m[/tex]

Now we multiply by the conversion factor in decimal form, placing the centimeters as numerator:

[tex]8.11\frac{N}{\operatorname{cm}}\times\frac{100\operatorname{cm}}{1m}=811.26\frac{N}{m}[/tex]

Therefore, the constant of the spring is 811.26 N/m.

part (b) Now we are asked to determine the distance is an object of 1.25 kg is place. First, we determine the weight of the new object:

[tex]W=mg[/tex]

Now we plug in the values:

[tex]W=(1.25\operatorname{kg})(9.8\frac{m}{s^2})=12.25N[/tex]

Now we use Hooke's law, but we solve for the distance "x" by dividing both sides by the constant "k", we get:

[tex]\frac{F}{k}=x[/tex]

Just as before, the only force acting is the weight, therefore, we plug in the values we got:

[tex]\frac{12.25N}{811.26\frac{N}{m}}=x[/tex]

Solving the operations:

[tex]0.015m=x[/tex]

Now we convert the meters into centimeters using the same conversion factor:

[tex]0.015m\times\frac{100\operatorname{cm}}{1m}=1.5\operatorname{cm}[/tex]

Therefore, the new mass stretches the spring 1.5 centimeters.

Part (c) Now we are asked to determine the work that has to be done on the spring to stretch it 8.9 centimeters. To determine that we will use the following formula for the work done on a spring:

[tex]w=\frac{1}{2}kx^2[/tex]

Now we replace the values:

[tex]w=\frac{1}{2}(811.26\frac{N}{m})(8.9\operatorname{cm})^2[/tex]

We will first convert the 8.9 centimeters into meters:

[tex]8.9\operatorname{cm}\times\frac{1m}{100\operatorname{cm}}=0.089m[/tex]

Now we replace this in the formula for the work:

[tex]w=\frac{1}{2}(811.26\frac{N}{m})(0.089m)^2[/tex]

Solving the operations we get:

[tex]w=3.21J[/tex]

Therefore, the work is 3.21 Joules.

At what speed must a satellite be travelling so that it shall remain in a circular orbit 1683049 m above the surface of the Earth. Take the mass of the Earth as 6.0 × 1024 kg

Answers

Given data

*The given mass of the Earth is M_e = 6.0 × 10^24 kg

*The distance above the surface of the Earth is r = 1683049 m

The formula for the speed of the satellite must be traveling so that it shall remain in a circular orbit is given as

[tex]v=\sqrt[]{\frac{GM_e}{r}}[/tex]

Substitute the known values in the above expression as

[tex]\begin{gathered} v=\sqrt[]{\frac{(6.67\times10^{-11})(6.0\times10^{24})}{(1683049)}} \\ =1.54\times10^4\text{ m/s} \end{gathered}[/tex]

Hence, the speed of the satellite must be traveling so that it shall remain in a circular orbit is v = 1.54 × 10^4 m/s

If it takes 0.8 s for your voice to be heard at a distance of 272 m, what is the temperature of the air?

Answers

V = 331 + 0.59 Tc

v= d/t = 272 / 0.8 = 340 m/s

340 = 331 + 0.59 Tc

340 - 331 = 0.59 TC

9 = 0.59 Tc

9/0.59 = Tc

Tc = 15.25°C

You have a coin sitting on
a card on top of a glass. You want to put the
coin into the glass, but you are not allowed
to pick up the card. Think of how you can do
that. Then write a short explanation of why
it works that would make sense to someone
who doesn't remember Newton's laws of
motion.

Answers

Answer:

You can flick the car to the side so that the coin can fall through.

Explanation:

What is physics and explain elements of physics

Answers

Physics is the branch of science that deals with mass , time , space, energy and their inter-relations .

The application of

Tsunami waves flood coastal and inland areas and affect coastal life. Which of these properties of tsunami waves most contributes to the flooding? The low friction of waves at the shoreline The high friction of waves at the shoreline The high amplitude of the waves at origination The huge volume of water that surges across shore

Answers

Take into account that the flooding produced by Tsunami waves has a great level of energy.

Moreover, consider that the amplitude of the waves determines the energy of the waves and then, the flooding is produced primarily by the high amplitude of the waves at origination.

How would you go about answering this question?

Answers

The new angular velocity becomes  0.286 rev/s.

Given parameters:

Mass of the marry go round: M = 120 kg.

Radius of the marry go round: r = 1.80 m.

Mass of the boy: m = 27 kg.

Initial angular velocity of the marry go round: ω₁ = 0.350 rev/s.

Final angular velocity of the marry go round: ω₂ = ?

From the principle of conservation of angular momentum;

Initial angular momentum = final angular momentum

⇒ 1/2 Mr²ω₁ = 1/2( M+m)r²ω₂

⇒ ω₂ = Mω₁/(m+M)

= 120×0.35/(120+27) rev/s

= 0.286 rev/s.

So, final angular velocity of the marry go round: ω₂ = 0.286 rev/s.

Learn more about angular velocity here:

https://brainly.com/question/12446100

#SPJ1

Equations of linear motion

Answers

The Newton's equations of motion are stated below

v = u + at

s = ut 1/2at^2

v^2 = u^2 + 2as

s = 1/2(u + v)t

where

u represents initial velocity

v represents final velocity

t represents time

a represents acceleration

The diagram below shows a block on a horizontal, frictionless surface. A 100.-newton force acts on the block at an angle of 30.0° above the horizontal. 100. N F 30.0° Block 77 Frictionless Surface What is the magnitude of force F, to the nearest tenth of a newton, if it establishes equilibrium?

Answers

ANSWER:

STEP-BY-STEP EXPLANATION:

Question one I need an explanation Pls. Only part a and b.

Answers

Given,

The velocity of the objects, v₁=50.0 m/s and v₂=-25.0 m/s

The initial positions of the objects, x₁_₀=0.00 m and x₂_₀=500 m

The relative velocity of the objects is given by,

[tex]v=v_1-v_2[/tex]

On substituting the known values,

[tex]\begin{gathered} v=50.0-(-25.0) \\ =75.0\text{ m/s} \end{gathered}[/tex]

The total distance between the objects is,

[tex]\begin{gathered} d=x_{10}+x_{20} \\ =0+500 \\ =500\text{ m} \end{gathered}[/tex]

(a)

As the two objects are traveling towards each other in a straight line, they will intercept.

The time it takes for the objects to intercept is given by,

[tex]t=\frac{d}{v}[/tex]

On substituting the know values,

[tex]\begin{gathered} t=\frac{500}{75} \\ =6.67\text{ s} \end{gathered}[/tex]

As t is the time it takes for the objects to intercept, the distance covered by the objects in this time will give us the position x_f where these two objects intercept.

As object 1 starts from the origin, the distance traveled by this object is equal to x_f

The distance traveled by object 1 in that time is,

[tex]x_f=d_1=v_1t[/tex]

On substituting the known values,

[tex]\begin{gathered} x_f_{}=50\times6.67 \\ =333.5m\text{ } \end{gathered}[/tex]

Thus the objects will intercept at the point x_f=333.5 m from the origin.

(b)

As calculated in part a, the time it takes for the objects to intercept is t=6.67 s

A ray of light is traveling through a mineral sample is submerged inwater. The ray refracts as it enters the water, as shown in the diagrambelow.NormalWater41°149°63°27°MineralCalculate the absolute index of refraction of the mineral.

Answers

We are asked to determine the absolute index of refraction of a mineral submerged in water. To do that we will use Snell's law:

[tex]n_1\sin \theta_1=n_2\sin \theta_2[/tex]

Where n1 and n2 are the refraction indices of water and mineral respectively and the angles "theta 1" and "theta 2" are the incidence and refraction angles. We will solve for n1:

[tex]n_1=\frac{n_2\sin \theta_2}{\sin \theta_1}[/tex]

Replacing the values:

[tex]n_1=\frac{(589.29nm)\sin 41}{\sin 27}[/tex]

Solving the operations:

[tex]n_1=851.58nm[/tex]

Therefore, the index of refraction of the mineral is 851.58 nm.

A ship's wheel has a moment of inertia of 0.930 kilogram·meters squared. The inner radius of the ring is 26 centimeters, and the outer radius of the ring is 32 centimeters. Disagreeing over which way to go, the captain and the helmsman try to turn the wheel in opposite directions. The captain applies a force of 314 newtons at the inner radius, while the helmsman applies a force of 290 newtons at the outer radius. What is the magnitude of the angular acceleration of the wheel?

Answers

We can use the formula of the moment of inertia given by:

[tex]r\cdot F=I\alpha[/tex]

Where:

r = Distance from the point about which the torque is being measured to the point where the force is applied

F = Force

I = Moment of inertia

α = Angular acceleration

So:

[tex]\begin{gathered} r\cdot F=(-0.26\times314+290\times0.32)=92.8-81.64=11.16 \\ I=0.930 \\ so,_{\text{ }}solve_{\text{ }}for_{\text{ }}\alpha: \\ \alpha=\frac{r\cdot F}{I} \\ \alpha=\frac{11.16}{0.930} \\ \alpha=\frac{12rad}{s^2} \end{gathered}[/tex]

Answer:

12 rad/s²

I need full explanation on how to solve both these questions I don't understand haha

Answers

Given that weight of 1 kg is equivalent to the weight of 2.2 lb.

The weight of one kg or 2.2 lbF is 9.8 N

(a)

The weight of 2.2 lbF is 9.8 N

Thus the force of 1.0 lbF is,

[tex]\begin{gathered} 1.0\text{ lbF}=\frac{1.0\text{ lbF}\times9.8\text{ N}}{2.2\text{ lbF}} \\ =4.45\text{ N} \end{gathered}[/tex]

Thus the force of the weight of 1.0 lbF is 9.8 N

(b)

If the thrusters are meant to use the value in N but used it in lbF, then the trusters would have used, say, 9.8 lbF in place of 9.8 N.

The force of 9.8 lbF is greater than the force of 9.8 N. Thus the force applied to slow the craft is higher than intended.

Thus they would slow the craft more.

(c)

Yes, the failure is correctly explained by the unit mix-up. As it is explained in part b, the force applied will be higher than intended. Which is exactly the case.

Thus the failure is correctly explained by the mix-up.

URGENT!! ILL GIVE
BRAINLIEST!!!! AND 100 POINTS!!!!!!

What type of energy transfer is shown in this image?

A. electrical to mechanical

B. chemical to electrical

C. electrical to chemical

D. chemical to mechanical

Answers

Answer:

The Correct Answer Is b.

_7. What happens to the total resistance of the resistors if they are connected inparallel? A. increases C. decreases B. remains the same D. increases then decreases

Answers

In order to find what happens to the total resistance when resistors are connected in parallel, let's analyze the formula below for the total resistance between two resistors in parallel:

[tex]\begin{gathered} \frac{1}{RT}=\frac{1}{R_1}+\frac{1}{R_2}\\ \\ RT=\frac{R_1R_2}{R_1+R_2} \end{gathered}[/tex]

When two resistors are in parallel, they have the same voltage.

That means more current will flow, because there will be current flowing from each resistor and the voltage is the same.

Calculating the total resistance, we will find a smaller resistance than the two resistors used, because with the same voltage, the current is greater.

Therefore the correct option is C.

1)A weight lifter raises a 180 kg barbell to a height of 1.95 m. What is the increase in the potential energy of the barbell? 2)A 20.0 kg rock is on the edge of a 100 m cliff. What is the velocity of the rock just before it hits the grounds when it is pushed off the cliff?

Answers

Question 1.

Given:

Mass of barbell = 180 kg

Height = 1.95 m

Let's find the increase in the potential energy of the barbell.

To find the potential energy, apply the formula:

PE = mgh

Where:

m is the mass = 180 kg

g is acceleration due to gravity = 9.8 m/s²

h is the height = 1.95 m

Input values into the formula and solve:

PE = 180 x 9.8 x 1.95 = 3439.8 J

Therefore, the increase in the potential energy of the barbell is 3439.8 J

ANSWER:

3439.8 J

Two billiard balls collide elastically ball A has a mass of 1.22 kg and an initial velocity of 0.8 M/S Ball B has a mass of 1.37 KG and an initial velocity of 0 M/S assuming that all of ball A’s is momentum is transferred to ball B after the collision what is the final velocity of Ball B ? *a perfect momentum transfer means that what was moving before the collision will not move after the collision and vice versa.A) 0.377 m/sB) 0.976 m/sC) 0.712 m/sD) 0 m/s

Answers

We will have the following:

[tex]\begin{gathered} (1.22kg)(0.8m/s)+(1.37kg)(0m/s)=(1.22kg)(0m/s)+(1.37kg)v_f \\ \\ \Rightarrow0.976kg\ast m/s=1.37kg\ast v_f\Rightarrow v_f=\frac{0.976kg\ast m/s}{1.37kg} \\ \\ \Rightarrow v_f=\frac{488}{685}m/s\Rightarrow v_f\approx0.712m/s \end{gathered}[/tex]

So, the final velocity will be approximately 0.712 m/s.

A standing wave is produced by reflecting a wave off a wall (which acts like a fixed end). If the standing wave consists of 6 anti-nodes and 7 nodes and the wall is 12m away from the source determine the wavelength.

Answers

Answer:

4 m

Explanation:

We can represent the standing wave as follows

So, we can divide the total distance into 6 parts as follows

12 m/6 = 2 m

Then, 2 m is the distance from node to node. It means that the wavelength will be twice this distance, so

wavelength = 2 x 2m = 4m

Therefore, the wavelength is 4m

A scenario where two people are sitting on a see saw is modeled here. Assuming both people are at equal distances from the pivot point, which statements below would result in the unbalanced forces shown above? Select ALL that apply.A)Person A has a force of 70 N and person B has a force of 80 N.B)Person A has a force of 80 N and person B has a force of 70 N.C)Person A has a force of 75 N and person B has a force of 75 N.D)Person A has a force of 50 N and person B has a force of 40 N.E)Person A has a force of 50 N and person B has a force of 70 N.

Answers

[tex]\begin{gathered} \text{From the pic, the person A must have more force than person B, hence} \\ \text{Stataments true:} \\ B)\text{ Person A has a force of 80N and person B has a force of 70N} \\ D)\text{Person A has a force of 50N and person B has a force of 40N} \end{gathered}[/tex]

A crate is at rest on an inclined plane. As the slope increases the crate remains at rest until the incline reaches an angle of 32.7° from the horizontal. At this angle the crate begins to slidedown the ramp.Apply Newton's 1st or 2nd law to each axis of the Free body diagram.

Answers

The free body diagram of the crate can be shown as,

According to free body diagram, the net force acting on the crate is,

[tex]F=mg\sin \theta-f\ldots\ldots\text{ (1)}[/tex]

The frictional force acting on the crate can be given as,

[tex]f=\mu N[/tex]

According to free body diagram, the normal force acting on the crate is,

[tex]N=mg\cos \theta[/tex]

Therefore, the frictional force becomes,

[tex]f=\mu mg\cos \theta[/tex]

According to Newton's second law, the net force acting on the crate is,

[tex]F=ma[/tex]

Therefore, equation (1) becomes,

[tex]\begin{gathered} ma=mg\sin \theta-\mu mg\cos \theta \\ a=g\sin \theta-\mu g\cos \theta \end{gathered}[/tex]

Thus, the net acceleration of the crate can be expressed using Newton's second law as,

[tex]a=g\sin \theta-\mu\cos \theta[/tex]

A cord is wrapped around the rim of a wheel 0.230 m in radius, and a steady pull of 37.0 N is exerted on the cord. The wheel is mounted on frictionless bearings on a horizontal shaft through its center. The moment of inertia of the wheel about this shaft is 4.60 kg⋅m2.

Answers

The angular acceleration of the wheel is 1.85m/s²

Acceleration is the rate at which speed and direction of velocity vary over time. Something is said to be accelerating when it starts to move faster or slower.

We are given that,

The pull force of the wheel = τ = 37.0N

The radius of the wheel = r = 0.230m

The moment of inertia = I = 4.60kg-m²

Here the angular acceleration = α = ?

The formula for angular acceleration can be given as,

α = τ /I

α = [(37.0N)×( 0.230m)]/4.60kg-m²

α = 1.85 m/s²

Hence , the angular acceleration of the wheel would be 1.85m/s²

To know more about acceleration

https://brainly.com/question/13925550

#SPJ1

Other Questions
Solve the equation using the Complete The Square Methodx^2+12x=13Do your work on paper. Take a picture, and upload it here. Show all your work/steps. 9. In April, Community Hospital reported 923 discharge days for adults and children and 107 discharge daysfor newborns. During the month, 192 adults and children and 37 newborns were discharged. Calculate theALOS for adults and children for the month of April. Round to one decimal place. At a sand and gravel plant, sand is falling off a conveyor and onto a conical pile at a rate of 4 cubic feet per minute. The diameter of the base of the cone is approximately three times the altitude. At what rate is the height of the pile changing when the pile is 22 feet high? Christopher ran for his life and came to a clearing just outside his house He and the dragon had run around in complete circles! He slumped on a rock and thought about his own family. All of a sudden,________________________________________Find a logical ending to this story Find the product of these complex numbers.(8 + 6i)(-5 + 7i) =A.-82 - 86iB.-82 + 26iC.2 + 26iD.2 - 86i The value of a ratio is 4/3. The second quantity in the ratio is how many times the first quantity in the ratio? William is 4 years older than three times Alex's age . William is 31 years old . How old is Alex Polygon ABCD with A (2,4), B (-4,-8), C (0,4), and D (12,-2), is dilated are the new coordinates? Is this a reduction or enlargement? Which line best illustrates the author's intent to show mr. phillips's ambivalence to anne's feelings? anne could run like a deer, however; run she did with the impish result that she overtook the boys at the door and was swept into the schoolhouse among them just as mr. phillips was in the act of hanging up his hat. mr. phillips's brief reforming energy was over; he didn't want the bother of punishing a dozen pupils; but it was necessary to do something to save his word, so he looked about for a scapegoat and found it in anne, who had dropped into her seat, gasping for breath, with a forgotten lily wreath hanging askew over one ear and giving her a particularly rakish and disheveled appearance. 'i assure you i did'"still with the sarcastic inflection which all the children, and anne especially, hated. it flicked on the raw. 'obey me at once.' when mr. phillips called the history class out anne should have gone, but anne did not move, and mr. phillips, who had been writing some verses 'to priscilla' before he called the class, was thinking about an obstinate rhyme still and never missed her. Does any one know what y-2x=5 The probability is(Round to four decimal places as needed.)Points: 0 of 1SaveAssume that when human resource managers are randomly selected, 43% say job applicants should follow up withintwo weeks. If 7 human resource managers are randomly selected, find the probability that at least 2 of them say jobapplicants should follow up within two weels. How does the creature's characterization change throughout the story?The character is defiant in the beginning, but becomes complacent at the endThe creature is hateful in the meginning, but learns compassion in the endThe creature loves humans in the beginning, but becomes indifferent to them atthe endThe creature is naive in the beginning, but becomes wise at the end f(x)=(0.13x+0.22x)-0.88x-0.25x-0.09for this polynomial use a graph and find the minimum and maximum values written as coordinates Problems 20 - 23. Analytically determine what type(s) of symmetry, if any, the graph of the equation would possess. Show your work.21) y^2 - xy = 6 Im an older lady not the best at this type of math please help Subtract 7x minus 8 from 2x^2- minus 1 what is the value of x in the equation 2.5 - 0.25x - -3 a blueprint for a house has a scale factor of 1 inch 3 ft. a wall in the blueprint is 5 in what is the length of the actual wall in feet Write 16 2/3% a as a decimal and b as a reduced fraction Ted and his classmates started watering flowers at 11:36 andfinished watering all the flowers at 13:25. If they wateredflowers at a constant rate, at what time did they finish watering4/7 of the flowers? Give your answer in a 24-hour clock format,such as 19:00.Enter the answer